0
$\begingroup$

Represent the following set of points in the XY plane:

{ (x,y) | |x|=1 }

{ (x,y) | |x| is less than or = 1 }

  • 1
    Where do you get stuck? Can you draw the set where $x = 1$ or $x = -1$?2012-06-09
  • 0
    yes its the answer for the first question right?2012-06-09
  • 0
    It is. Now for the second one, this also includes lines such as $x = 1/2$, $x = -1/3$ or more generally lines $x = a$ for every $a$ with $-1 \le a \le 1$. Can you draw it now?2012-06-09
  • 0
    for the second one, will the line start from -1 till 1 on the x axis2012-06-09
  • 0
    You may want to read about [why should we accept answers](http://meta.math.stackexchange.com/questions/3399/why-should-we-accept-answers) and [how do we do that](http://meta.math.stackexchange.com/questions/3286/how-do-i-accept-an-answer/3287#3287). Your accept ratio is less than 40% which is less than acceptable on this site.2012-06-09
  • 0
    hey sorry i will remember this for the next time, actually i had not much information about that..2012-06-09

2 Answers 2

1

HINT: Note that your set imposes no restriction at all on the $y$-coordinate. If you find a point in the set, every other point with the same $y$-coordinate will also be in the set. Now, what does $\{x\in\Bbb R:|x|\le 1\}$ look like as a subset of the real line?

Added: Your region is the blue stripe in the picture below; it extends infinitely far up and down. enter image description here

  • 0
    for the second one, will the line start from -1 till 1 on the x axis2012-06-09
  • 0
    @meg_1997: That’s right, and it will include both endpoints, $-1$ and $1$. Thus, your region in the plane will be an infinite vertical stripe running between the lines $x=-1$ and $x=1$.2012-06-09
  • 0
    is it possible to show it on graph here by you2012-06-10
  • 0
    @meg_1997: I can produce a rough picture; hang on for a few minutes while I do it.2012-06-10
  • 0
    hey thanks for that:)2012-06-10
1

Hint: Can you represent them on the $x$ axis? Note that since the criteria do not depend on $y$ the sets will be vertical lines at all $x$ that are acceptable.